A movement that keeps the distal end of the body segment fixed in one location describes what type of kinetic chain movement? Open kinetic chain movement Lateral kinetic chain movement Dynamic kinetic chain movement Closed kinetic chain movement

Answers

Answer 1

A movement which keeps the distal end of the body segment fixed in one location describes what type of kinetic chain movement is known as a closed kinetic chain movement.

What is a closed kinetic chain movement?

A closed kinetic chain movement is defined as that position where the most distal aspects of a particular extremity are fixed to the earth or another solid object.

In conclusion; a movement which keeps the distal end of the body segment fixed in one location describes what type of kinetic chain movement is known as a closed kinetic chain movement.

Learn more about closed kinetic chain motion:

https://brainly.com/question/12405822

#SPJ1


Related Questions

calculate the kinetic energy of 200kg object that is moving with a speed 15ms.

Answers

Answer:

See below

Explanation:

Kinetic Energy = 1/2 * m * v^2

                        = 1/2 * 200 * (15)^2 = ________ joules

The kinetic energy of the 200 kg object moving with a speed of 15 m/s is 11250 Joules (J).

Given:

Mass of the object (m) = 200 kg

Speed of the object (v) = 15 m/s

For the kinetic energy (KE) of an object, we use the formula:

KE = (1/2) × m × v²

where:

KE is the kinetic energy,

m is the mass of the object,

v is the speed of the object.

The kinetic energy of the object:

KE = (1/2) × 200 × (15 )²

KE = (1/2) × 200 × 225

KE = 11250

KE = 11250 Joules (J)

Therefore, the kinetic energy of the 200 kg object moving with a speed of 15 m/s is 11250 Joules (J).

To know more about kinetic energy:

https://brainly.com/question/30107920?referrer=searchResults

#SPJ3

from the same height (and at the same time), one ball is dropped and another ball is fired horizontally. which one will hit the ground first

Answers

Both balls will take the same amount of time to hit the ground if they are dropped and fired from the same height.

What is Projectile motion:

It is the motion of an object thrown or projected into the air, subject to only the acceleration of gravity. The object is called a projectile, and its path is called its trajectory.

For a ball fired horizontally from same height and time, there is only a vertical acceleration on it towards the ground.

This acceleration is equal to the acceleration due to gravity (g = 9.81 m/s^2).

A ball dropped from the same height will also experience the same vertical acceleration downwards as the horizontal acceleration.

Therefore,

Both balls will take the same amount of time to hit the ground if they are dropped and fired from the same height.

Learn more about Projectile motion here:

https://brainly.com/question/11049671

#SPJ4

If the red and green balls fall off opposite sides of the table, the green ball will make a louder
sound. why?
select one:
o a. the green ball has more potential energy, meaning it had less kinetic energy.
potential energy equals sound energy.
o b. the green ball has more mass, so it had more kinetic energy. the kinetic energy
transferred to sound energy.
o c. the green ball was moving at a faster rate, and speed of movement determines the
loudness of the sound.
od. the red ball has less potential energy, so the green ball had more potential energy.
as energy transferred from one ball to the other, sound was created.

Answers

B. The green ball has more mass, so it had more kinetic energy, the kinetic energy transferred to sound energy.

What is kinetic energy?

This is the energy possessed by a moving object, and it depends on mass and speed of the object.

K.E = 0.5mv²

where;

m is mass of the objectv is the speed of the object

Thus, we can conclude that, the green ball has more mass, so it had more kinetic energy, the kinetic energy transferred to sound energy.

Learn more about kinetic energy here: https://brainly.com/question/25959744

#SPJ1

If the strength of the earth's electric field is 100 N/CN/C , how does the magnitude of the electric force on the droplet compare to the weight force

Answers

Magnitude of electric force = 15 x 10⁻¹⁰ N

Magnitude of weight force = 64.092 x 10⁻⁶ N

Given

Diameter of rain drop = 2.5mm

Radius = 1.25 mm = 1.25 × 10⁻³ m/sec

Drop has a charge = +15pC

15pC = 15×10⁻¹²C

Electric field (E) = 100 N/C

Electric force on the charge

= charge x electric field = qE

= 15 x 10⁻¹² x 100

= 15 x 10⁻¹⁰ N.

This force will act in upward direction as the force on the positive charge will always acts in the direction of the electric field.

Volume of rain droplet(V) = 4/3 π R³

Volume V = 4/3 x 3.14 x ( 1.25 x 10⁻³)³

                 = 6.54 × 10⁻⁹m³

As Density of water = 1000 kg / m³

Density =Mass/Volume

So, Mass of rain droplet = 1000 x 6.54 x 10⁻⁹ kg

= 6.54 x 10⁻⁶ kg .

Now, Weight(W) = mg

= 6.54 x 10⁻⁶ x 9.8

= 64.092 x 10⁻⁶ N.

As the electric force is 15 × 10⁻¹⁰ N

So, weight force is more than the electric force.

Hence, the magnitude of the electric force on the droplet is less as compare to the weight force.

Learn more about Electric force & weight here https://brainly.com/question/13258339

#SPJ4

(picture attached) use grass method
4. A block of unknown substance is submerged in water. A light ray in the water strikes the angle of 16°, what is the index of refraction of the unknown substance? (nwater = 1.33) substance at an angle of 45° from the normal. If the refracted ray in the substance is at an 5. If a light ray passes from a substance with low index of refraction to another substance with high index of refraction, will the ray bend away from or closer to the normal? Why? 6. What is the angle of refraction? n₁=1 24° n₂ = 1.8​

Answers

Answer:

Explanation:

Step 1

1 of 2

\textbf{Given}Given

n_{w}=1.33n

w

=1.33

$\theta_w=31\text{ ^\circ

}$

$\theta_b=27\text{ ^\circ

}$

\textbf{Approach}Approach

In this problem we are going to use Snell's law.

\textbf{Solution}Solution

The definition of Snell's law of refraction is

\begin{align} {n_1\cdot\sin \theta_1}={n_2\cdot \sin \theta_2} \end{align}

n

1

⋅sinθ

1

=n

2

⋅sinθ

2

 

where indexes 11 and 22 represent two different mediums. Since the motion is in the water we write

\begin{align} &{n_{w}\cdot\sin\theta_{w}}={n_{b}\cdot\sin\theta_{b}} \\ &{n_b}={n_{w}\cdot \frac{\sin\theta_{w}}{ \sin \theta_{b}} } \\ &{n_b}={1.33\cdot \frac{\sin31^\circ}{ \sin27^\circ} } \\ &\boxed{{n_b}=1.5} \end{align}

 

n

w

⋅sinθ

w

=n

b

⋅sinθ

b

n

b

=n

w

sinθ

b

sinθ

w

n

b

=1.33⋅

sin27

sin31

n

b

=1.5

What is the value of the current intensity that crosses a wire of 10 ohms if the voltage applied to the extremes of the wire is 110 V

Answers

11 A is the amount of current that crosses the wire.

According to Ohm's law,

If all other physical parameters and temperatures remain constant, the voltage across a conductor is directly proportional to the current flowing through it.

V ∝ I

V= IR where,

V = Voltage across a conductor

I = Current flowing through the conductor

R= Resistance and also known as constant of proportionality

Given,

Resistance of wire(R)  = 10 ohms

Voltage applied(V) = 110V

Current(I) = ?

According to the formula

110V  = I × 10Ω

I = 110/10 = 11 A

Hence, current intensity  of 11 A crosses a wire of 10 ohms if the voltage applied to the extremes of the wire is 110 V

Learn more about Ohm's law here https://brainly.com/question/14296509

#SPJ4

Using the information from the previous problem (A 2kg ball rotates on the end of a 1.4m long string. The ball makes 5 revolutions in 4.4s.), calculate the centripetal force on the ball.
Group of answer choices

A) 140.9N

B) 1.1N

C) 110.4N

D) 5.7N

Answers

The centripetal force on the ball is 140.9 N; option A

What is the centripetal force on the ball?

The centripetal force on the ball is given by the formula below:

F = mv²/r

where;

m is massv is linear velocity r is radius

The linear velocity, v = wr

v = 5 * 2π/4.4 * 1.4 = 9.93 m/s

F = 10² * 2/1.4 = 140.9 N

In conclusion, the centripetal force is derived from the mass, velocity and radius of the path.

Learn more about centripetal force at: https://brainly.com/question/20905151

#SPJ1

A box with a mass of 2 kg only has four forces acting on it: One force of 16 N due East. One force of 24 N due South. One force of 16 N due West. One force of 18 N due North. What is the magnitude of the box's acceleration, in m/s2

Answers

Answer:

The acceleration of the object is 3 m/s2.

Explanation:

The force on the object is acting along the four directions which are perpendicular to each other. Since the north and south directions are opposite to each other, the magnitude of the net force along north-south direction is (24-18)=6 N.Since the east and west directions are opposite to each other, the magnitude of the net force along east-west direction is (16-16)=0 N.Therefore the magnitude of the net force on the object is 6 NBy Newton's second law, the force F on the object is given by the formula F=ma. Here m= 2kg, therefore 6 =(2)a which gives a = 3 m/s2.        

Learn more about Newton's law.

brainly.com/question/13447525

#SPJ10

How can we change direction of light? name this phenomenon. grade 6 please helppp

Answers

The direction of the light can be changed by reflection from a reflecting surface and by the refraction of the light through a medium of different refractive index.

What is reflection?

The phenomenon of the light in which the light ray bounces back from the surface on which they fall is called the reflection.

Now when the light ray falls on a surface that is well polished like the mirror, then the light ray changes its direction. So by using the combination of mirrors, the direction of the light ray can be changed.

What is refraction?

Refraction is a phenomenon in which light rays change their path slightly when it is passing through a transparent medium with a different refractive index.

The concave and convex lens uses this property of light. So by using the convex and concave lens, the direction of the light ray can be changed.

Learn more about refraction.

https://brainly.com/question/13088981

#SPJ4

A 480 g peregrine falcon reaches a speed of 75 m/s in a vertical dive called a stoop. If we assume that the falcon speeds up under the influence of grav- ity only, what is the minimum height of the dive needed to achieve this speed

Answers

The minimum height of the dive needed to achieve the given speed is v = 69 m/s is 242.9 m.

Given information:

The mass of peregrine falcon is,  m = 480

The final speed reached by the peregrine falcon in a vertical dive is, v = 69 m/s

It is given that the falcon is diving vertically downward. It can be compared with the same situation as the free-falling object under the effect of gravity only. So, the initial velocity of the falcon will be u = 0 m/s  as the motion starts with rest.

The value of the gravitational acceleration of gravity is, g = 9.80 m/s²

Now, using the third equation of motion, the minimum height  required for the final speed will be,

v² - u² = 2gh

69² - 0² = 2 × 9.8 × h

h = 242.9m.

Therefore, the minimum height of the dive needed to achieve the given speed is 242.9 m.

Learn more about falcon speeds at

https://brainly.com/question/12449855

#SPJ4

Flexible cord sets and power-supply cords shall not be concealed behind building __________________, or run through doorways, windows, or similar openings.

Answers

Flexible cord sets and power-supply cords shall not be concealed behind building , structural ceilings, suspended or dropped ceilings, floors or walls, or run through doorways, windows, or similar openings.

Hence Option (b) All of these is correct

Flexible cord and flexible cable usage are now outlined in Section 400.10, while prohibited uses are listed in Section 400.12.

Note 5 under 400.12 now includes a new exclusion for flexible wires and cords (uses not permitted).

Flexible cables, flexible cord sets, and power supply cords are not permitted in areas that are "concealed by walls, floors, or ceilings or positioned above suspended or dropped ceilings," according to Note 5.

The following uses for flexible cables, flexible cord sets, and power supply cords are prohibited as well:

(1) As an alternative to a structure's fixed wiring.

(2) Run-through openings in structural ceilings, floors, suspended ceilings, or dropped ceilings.

(3) When run through windows, doorways, or comparable openings.

(4) If attached to building surfaces.

Hence, Option (b) All of these is correct .

Learn more about Flexible cords here https://brainly.com/question/3950810

#SPJ4

Which arrow represents the substance’s change of state?

Answers

Answer:

Its P path describes the change of state i.e from solid to liquid.

What is solid ?

"A solid is that state of matter which has a fixed shape, mass, and volume. It suffers very small changes in volume by changing the temperature. It can not be compressed, e.g. — Sand, Wood, Copper, Ice, etc."

What is liquid ?

"A liquid is a sample of matter that conforms to the shape of a container in which it is held, and which acquires a defined surface in the presence of gravity. The term liquid is also used in reference to the state, or condition, of matter having this property."

A glass optical fiber is used to transport a light ray across a long distance. The fiber has an index of refraction of 1.510 and is submerged in water, which has an index of refraction of 1.333. What is the critical angle (in degrees) for the light ray to remain inside the fiber

Answers

The answer is 73.13°.

According to snell's law,

n1sinθi = n2sinθr

n1/n2 = sinθr/sinθi

The critical angle is the angle of incidence at the denser medium when the angle of incidence at the less dense medium is 90°

This means i=C and r = 90°

The Snell's law formula will become

n1/n2 = sinC/sin90°

n2/n1 = 1/sinC

Where n1 is the refractive index of the less dense medium = 1.473

n2 is the refractive index of the denser medium = 1.540

Substituting the values in the formula,

1.540/1.473 = 1/sinC

1.045 = 1/sinC

SinC = 1/1.045

SinC = 0.957

C = sin^-1(0.957)

C = 73.13°

The refractive index of glass is 1.5. It way that the velocity of mild in glass is 1.5 times slower than the rate of mild in a vacuum, the speed of light in glass isn't always unbiased of the shade of mild.

Refractive index is likewise the same as the velocity of light c of a given wavelength in an empty area divided with the aid of its speed v in a substance or n = c/v.

The index of refraction of fabric is a ratio that compares the velocity of light in a vacuum ( c=3.00x108ms ) to the velocity of mild in that precise medium. Because the index of refraction increases, the amount that the material bends the mild increases.

Learn more about the index of refraction here https://brainly.com/question/12469161

#SPJ4

The critical angle (in degrees) for the light ray to remain inside the fiber is 73.13°.

According to snell's law,

n1sinθi = n2sinθr

n1/n2 = sinθr/sinθi

The critical angle is the angle of incidence at the denser medium when the angle of incidence at the less dense medium is 90°.

This means i=C and r = 90°

The Snell's law formula will become

n1/n2 = sinC/sin90°

n2/n1 = 1/sinC

Where n1 is the refractive index of the less dense medium = 1.473

n2 is the refractive index of the denser medium = 1.540

Substituting the values in the formula,

1.540/1.473 = 1/sinC

1.045 = 1/sinC

SinC = 1/1.045

SinC = 0.957

C = sin^-1(0.957)

C = 73.13°

Learn more about critical angle here https://brainly.com/question/15009181

#SPJ4

If an electron has an uncertainty in its velocity of 1.40 m/s, what is the uncertainty (in meters) in its position

Answers

Uncertainty in position:

If the electron has an uncertainty in its velocity of 1.40 m/s then the uncertainty in its position is [tex]4.14\times10^{-4} \text{ m}[/tex].

Heisenberg's uncertainty principle to calculate the required:

Step-1:

We have to apply Heisenberg's uncertainty principle to calculate the uncertainty in the position of an electron. According to the principle:

[tex]$\Delta x \Delta p \geq \frac{h}{4 \pi}$[/tex]

Here,

[tex]$\Delta x$[/tex] is the uncertainty on the position measurement

[tex]$\Delta p$[/tex] is the uncertainty on the momentum measurement

h is the Planck constant.

It is known that the momentum of a particle is calculated as, the product of the mass of the particle and its velocity.

Therefore,

p=mv

Thus the Heisenberg principle becomes:

[tex]$m \Delta x \Delta v \geq \frac{h}{4 \pi}$[/tex]

Here [tex]$\Delta v$[/tex] is the uncertainty in the velocity measurement.

It is given that, [tex]$\Delta v$[/tex]=1.40 m/s. The particle is electron here, thus the mass of the particle m=[tex]9.1 \cdot 10^{-31} \mathrm{~kg}[/tex] and the value of the Plank's constant is, h=[tex]6.62 \times 10^{-34} \mathrm{~m}^{2} \mathrm{~kg} / \mathrm{s}[/tex]

Step-2:

Substituting the values into the equation to get the value of the position uncertainty.

[tex]\Delta x \geq \frac{h}{4 \pi\times m\times \Delta v}\\\geq\frac{6.62\times10^{-34}}{4\pi \times 9.1\times10^{-31}\times1.40} \text{ m}\\\geq 4.14\times10^{-4} \text{ m}[/tex]

To know more about the uncertainty in position, refer to:

https://brainly.com/question/9574825

#SPJ4

Two workers are sliding 440 kg crate across the floor. One worker pushes forward on the crate with a force of 390 N while the other pulls in the same direction with a force of 260 N using a rope connected to the crate. Both forces are horizontal, and the crate slides with a constant speed. What is the crate's coefficient of kinetic friction on the floor

Answers

The kinetic coefficient of friction of the crate is 0.1506

Let's assume that both workers are applying positive pressure to the crate. A body cannot change its state of motion whether it is at rest or moves uniformly, according to Newton's First Law (at constant velocity). The magnitude of the friction force must therefore equal the total of the two external forces. The crate's equilibrium equations are as follows:

∑Fₓ = P + T - μ . N = 0

[tex]F_{y}[/tex] = N - W = 0

Where:

[tex]P[/tex] - Pushing force, measured in newtons.

[tex]T[/tex] - Tension, measured in newtons.

μ- Coefficient of kinetic friction, dimensionless.

[tex]N[/tex]- Normal force, measured in newtons.

W - Weight of the crate, measured in newtons.

The system of equations is now reduced by algebraic means:

P + T - μ . W = 0

We finally clear the coefficient of kinetic friction and apply the definition of weight :

μ [tex]=\frac{P + T}{m.g}[/tex]

We know that P = 390 N, T = 260 N, m = 440 kg and [tex]g= 9.807 \frac{m}{s^2}[/tex] ;

μ = 0.1506

Therefore, the kinetic coefficient of friction of the crate is 0.1506

Learn more about kinetic friction here:

https://brainly.com/question/19392943

#SPJ4

A series combination of a 48.0-mF capacitor and a resistor are connected to a 12-V battery. After one second, the voltage across the capacitor is 10 V. What is the resistance of the resistor

Answers

The resistance of the resistor is .

Given:-  A series combination of a 48.0-mF capacitor and a resistor are connected to a  battery. After one second, the voltage across the capacitor is .

To Find:- We have to find the value of the resistance of the resistor.

By using the concept of the resistance it will be solved.

According to the problem,

The value of the resistance of the resistor is .

#SPJ4

A stone is dropped from the top of a cliff. It hits the ground below after 3.25 s. How high is the cliff

Answers

The height of the cliff above the ground is 51.76 m.

Height of the cliff

h = vt + ¹/₂gt²

where;

v is initial vertical velocityt is time of motiong is acceleration due to gravity

h = 0 + ¹/₂gt²

h = ¹/₂gt²

h = ¹/₂(9.8)(3.25)²

h = 51.76 m

Thus, the height of the cliff above the ground is 51.76 m.

Learn more about height here: https://brainly.com/question/1739912

#SPJ1

How does light travel? (1 Point)
A) "Light can only travel in a vacuum and the speed varies depending on whether the source is moving or stationary"
B) "Light can travel in the air or a vacuum and the speed is constant whether the light source is stationary or moving"
C) "Light only travels in a medium and light travels when a sound source is stationary"
D) "Light only travels in a vacuum and travels faster when the sound source is moving"

Answers

Light can travel in the air or a vacuum and the speed is constant whether the light source is stationary or moving which is denoted as option B.

What is Light?

This is referred to the portion of the electromagnetic spectrum which is perceived by the human eyes.

It has a constant speed and can travel in an air or vacuum. This theefrore makes it the most appropriate choice ion this scenario.

Read more about Light here https://brainly.com/question/1363382

#SPJ1

Which best describes a possible transfer of energy

Answers

A mechanical wave transfers energy through matter.

What is Mechanical wave?

A mechanical wave is a wave that is an oscillation of matter and is responsible for the transfer of energy through a medium.

Answer: Like all waves, mechanical waves transport energy. This energy propagates in the same direction as the wave. A wave requires an initial energy input; once this initial energy is added, the wave travels through the medium until all its energy is transferred. In contrast, electromagnetic waves require no medium, but can still travel through one.

A wave is a transmission of a disturbance from one point to another. Wave involves transmission of energy from one point to another through a materials medium.

Mechanical waves vibrate in matter and transfer energy. Explore the production and propagation of mechanical waves and discover the factors that affect mechanical waves including elasticity, tension, temperature, and density

Therefore,

A mechanical wave transfers energy through matter.

A complete question is is given below:

What best describes a possible transfer of energy?

a. A mechanical wave transfers energy through matter.

b. A longitudinal wave transfers energy perpendicular to wave motion.

c. A transverse wave transfers energy parallel to wave motion.

d. A sound wave does not need a medium to transfer energy through.

Learn more about mechanical wave here:

https://brainly.com/question/3101711

#SPJ1

if a man pulls a box with a forcce of 90N at an angle of 45 degrees then what are x and y components of the force

Answers

The x and y components of the force will be 63.6 N and 63.6 N  respectively.

What is velocity?

The change of distance with respect to time is defined as speed. Speed is a scalar quantity. It is a time-based component. Its unit is m/sec.

The given data in the problem is

Force, F=90 N

Angle,θ = 45°

The force of 90N at an angle of 45 degrees;

The force is resolved in the x and y direction as;

[tex]\rm F_x = 90 cos 45 ^ 0 \\\\ F_x = 63.6 \ N[/tex]

[tex]\rm F_y = 90 sin 45^0 \\\\ F_y =63.6 \ N[/tex]

Hence x and y components of the force will be 63.6 N and 63.6 N  respectively.

To learn more about the velocity, refer to the link: https://brainly.com/question/862972.

#SPJ1

When the liquid line is restricted, the supply of refrigerant to the metering device is reduced. What is the effect on suction pressure and superheat

Answers

Answer:

The suction pressure decreases and the superheat increases when the liquid line is restricted and the supply of refrigerant to the metering device is reduced.

Explanation:

1. The five components of refrigeration are:

Fluid refrigerantCompressorCondenser coilEvaporator coilExpansion device.

       The compressor limits the vapor released by the refrigerant. This            

       causes a rise in pressure (in refrigerant), which then pushes the  

       vapor into the coils on the outside of the refrigerator.

2. Now when the cooler air meets the warm gas present in the coils, it

   gets converted into liquid form.

3. Thus, when the liquid form is at high pressure, the refrigerant then  

   cools down as it flows through the coils placed in the fridge ( in both

   freezing and normal sections).

4. The refrigerant also absorbs the warm air present in the fridge, which  

   causes it to evaporate and flow back through the compressor and the

   cycle repeats in the same form.

Thus, when the liquid line is restricted and the supply of refrigerant to the metering device is reduced it causes a decrease in suction pressure and an increase in superheat.

Learn more about refrigeration here:

https://brainly.com/question/9046279

#SPJ4

Two objects, m1 = 0.6 kg and m2 = 4.4 kg undergo a one-dimensional head-on collision
Their initial velocities along the one-dimension path are vi1 = 32.4 m/s [right] and vi2 = 8.6 m/s [left].
The two objects stick together after the perfectly inelastic collision
a) Calculate the velocity after the collision.
b) Determine how much kinetic energy is lost due to the collision

Answers

Explanation:

Hello !

look at the attachment above ☝️ and if you have any questions your welcome.

a) The velocity after the collision.is 11.456 m/s.

b) The kinetic energy lost due to the collision is 44.564 J.

What is conservation of momentum principle?

When two bodies of different masses move together each other and have head on collision, they travel to same or different direction after collision.

The external force is not acting here, so the initial momentum is equal to the final momentum. For inelastic collision, final velocity is the common velocity for both the bodies.

m₁u₁ +m₂u₂ =(m₁ +m₂) v

Given are the two objects, m1 = 0.6 kg and m2 = 4.4 kg undergo a one-dimensional head-on collision. Their initial velocities along the one-dimension path are vi1 = 32.4 m/s [right] and vi2 = 8.6 m/s [left].

(a) Substitute the values, then the final velocity will be

0.6 x32.4 +4.4 x 8.6 = (0.6+4.4)v

v = 11.456 m/s

Thus, the velocity after collision is 11.456 m/s.

(b) Kinetic energy lost due to collision will be the difference between the kinetic energy before and after collision.

= [1/2m₁u₁² +1/2m₂u₂² ] - [1/2(m₁ +m₂) v²]

Substitute the value, we have

= [1/2 x 0.6 x32.4² + 1/2 x4.4 x 8.6²] - [1/2 x(0.6+4.4)11.456²]

= 44.564 J

Thus, the kinetic energy lost due to the collision is 44.564 J.

Learn more about conservation of momentum principle

https://brainly.com/question/14033058

#SPJ2

To develop a model of blackbody radiation that correctly modeled the behavior of emission intensity versus wavelength, Planck applied quantization of:\

Answers

Max Planck utilized the concept of energy quantization in order to develop a model for blackbody radiation that accurately fit experimental results.

Electromagnetic waves emitted by using a blackbody are referred to as blackbody radiation. A blackbody is bodily found out through a small hole within the wall of a hollow space radiator.

The depth of blackbody radiation depends on the wavelength of the emitted radiation and the temperature T of the blackbody.

When the temperature of a black frame increases, it is discovered that the wavelength similar to most electricity changes from 0.26 μm to 0. Thirteen μm Then the ratio of the emissive power of the body at the respective temperature is.

Learn more about Electromagnetic waves here: https://brainly.com/question/25847009

#SPJ4

After rubbing on the apparatus, your pith pall became negatively charged. What type of charge does the apparaTapus now have

Answers

         The apparatus must be Positively Charged.

Charge interaction:

    As from the charge interaction we came to know that when we   rub two objects together then there will be three possibility.

        1. opposite charge objects attract each other

        2. like charge object repel each other

        3. neutral object with any charges will attract each other.

    These happens due to friction generated on rubbing of object which in reverse create electron and get transfer.

  So, In the question it said that after rubbing on apparatus pith ball

   become negative charge which means it has gained the electron.

          Thus, the apparatus must have lose electron and would be

          positively charged.

   

          Learn more about pith ball here:

                     https://brainly.com/question/1131516

                       #SPJ4

 

       

What is the current flowing through the circuit shown? (V= 120 V, R₁ = 20 02,
R₂= 50 Q, R3= 1022) (Ohm's law: V = IR)
R₁
ww
ww
R₂
ww
R₂

Answers

Assumed that resistors are connected in series .

R_net=20+50+10=80ohm

Voltage=V=120V

Current=I

V/R120/803/21.5A

_____________is the addition of wave energy as waves interact, producing larger waves and ____________ is the subtraction of wave energy as waves interact, producing smaller waves.

Answers

Constructive interference is the addition of wave energy as waves interact, producing larger waves and destructive interference is the subtraction of wave energy as waves interact, producing smaller waves.

To find the answer, we need to know about the interference of waves.

What's the interference of waves?Interference of waves is the result of superposition of waves (transverse or longitudinal) at a certain place. Interference is of two typesConstructive interferenceConstructive interferenceDestructive interferenceHow are the constructive and destructive interference formed?When two waves are superimposed within the phase, then constructive interference pattern is formed.When two waves are superimposed with out of phase, destructive interference pattern is found.

Thus, we can conclude that the constructive interference is the addition of wave energy and destructive interference is the subtraction of wave energy.

Learn more about the interference of waves here:

brainly.com/question/12466679

#SPJ4

Answer the following questions.
1. What is the velocity of a car that traveled a total of 75 kilometers north in 1.5 hours?
2. What is the velocity of a plane that traveled 3,000 miles from New York to California in 5.0 hours?
3. John took 45 minutes to bicycle to his grandmother's house, a total of four kilometers. What was his speed in km/hr?.
4. It took 3.5 hours for a train to travel the distance between two cities at a speed of 120 miles/hr. How many miles lie between the two cities?​

Answers

3.if time=45mins,D=4km,2700secs,4000m then. S=d/t S=1.4m/s

Directly detecting an Earth-size planet around one of the nearest stars (besides the Sun) is roughly equivalent to trying to take a picture of a ball point from a pen at a distance of about ________ kilometers.

Answers

The ballpoint from a pen at a distance of about  4000  kilometers.

Alpha Centauri is the closest star device. Distance from Earth 4,365 al (forty 000 billion kilometers. The solar (yellow famous person) distance from Earth is 150 000 000 km, the exoplanet distance from Alpha Centauri b (orange famous person) 6 000 000 km.

Shifts in starlight from Proxima Centauri, found over extra than 2 years, screen its 1/3 planet. Astronomers have observed a 3rd planet orbiting Proxima Centauri, the star closest to the solar.

Proxima Centauri, the nearest megastar to our own, remains forty,208,000,000,000 km away. (Or about 268,770 AU.)

Learn more about the nearest stars here https://brainly.com/question/21107590

#SPJ4

Point charges q1and q2 of +12nc and -12nc,respectively are placed 10.0cm apart.compute the resultant electric field(magnitude field)

Answers

The electric field due to charge q1 and q2 at the middle point of the line joining the charges q1 and q2 is 8.64*10^(4) N/C.

Note: You have not given the point where the resultant magnetic field has to be calculated. Most probably it is asked to find the electric field at the middle point of the line joining the point charges.

Electric field: The electric force exerted on a unit charge is called the electric field. Electric field due to a charge is calculated using the formula,

E=kq/r^2

where k is a constant whose value is 9*10^(9) N m^2/ C^(2), q is the charge and r is the distance from the charge to the point where the electric field has to be calculated. In the given case, the electric field has to be calculated at the middle point of the line joining charges q1 and q2.

Calculation of electric field due to charge q1:

Given that q1=+12 nC or q1=+12*10^(-9) C and the distance of the charge q1 from the center d1=10.0/2 cm or d1=5*10^(-2) m, the magnitude of the electric field E1 due to charge q1 is,

E1=kq1 /d1^(2)

E1=9*10^(9)*12*10^(-9) / (5*10^(-2))^2

E1=4.32*10^(4) N/C

The direction of the electric field E1 at the middle point is towards the negative charge.

Calculation of electric field due to charge q2:

Given that q2=-12 nC or q1=-12*10^(-9) C and the distance of the charge q2 from the center d2=10.0/2 cm or d2=5*10^(-2) m, the magnitude of the electric field E2 due to charge q2 is,

E2=kq2 /d2^(2)

E2=9*10^(9)*12*10^(-9) / (5*10^(-2))^2

E2=4.32*10^(4) N/C

The direction of the electric field E2 at the middle point is towards the negative charge.

Total electric field:

The total electric field is given by the addition of the electric field. The direction of the electric field is the same for both charges, hence total electric field E is,

E=E1+E2

E=4.32*10^(4) +4.32*10^(4)

E=8.64*10^(4) N/C

Learn more about electric field,

https://brainly.com/question/9699355

#SPJ4

You look in the sky and see two jetliners that you know are of equal size, yet one appears to be much larger. Because of your knowledge of __________, you will assume that the smaller jetliner is farther away

Answers

Answer:

Because of the knowledge of relative size, it will be assumed that the smaller jetliner is farther away.

Explanation:

According to the theory of relative size, the distance that an object has to the viewing individual affects the perception of the individual regarding the size of the object.

As stated in this case, one of the jetliners is farther away from the other. Therefore, even if the jets are of equal size, the one that is at a greater distance is perceived to be smaller as it is at a greater viewing range. The one that is nearer to the individual seems bigger in comparison to the one farther away due to a closer viewing range.

Therefore, the jet that is nearer appears larger.

To know more about relative size, refer to:

https://brainly.com/question/19998265

#SPJ4

Other Questions
Figure ABCD is transformed to figure A prime B prime C prime D prime, as shown below:A coordinate grid is shown from negative 5 to 0 to 5 on both x -and y-axes. A polygon ABCD has A at ordered pair 1, 3, B at ordered pair 3, 4, C at ordered pair 2, 1, D at ordered pair 1, 1. A polygon A prime B prime C prime D prime has A prime at ordered pair negative 3, 3, B prime at ordered pair negative 5, 4, C prime at ordered pair negative 4, 1, D prime at ordered pair negative 3, 1.Which of the following sequences of transformations is used to obtain figure A prime B prime C prime D prime from figure ABCD? Reflection over the x-axis followed by a translation to the right by 2 units Reflection over the y-axis followed by a translation to the left by 2 units Counterclockwise rotation by 90 degrees about the origin followed by a translation to the right by 2 units Counterclockwise rotation by 90 degrees about the origin followed by a translation to the left by 2 units How does the area of triangle rst compare to the area of triangle lmn? the area of rst is 2 square units less than the area of lmn. the area of rst is equal to the area of lmn. the area of rst is 2 square units greater than the area of lmn. the area of rst is 4 square units greater than the area of lmn. Solve the equation f(x)=x3+2x2+4x+8 given that one of its roots is x=2. what is the formula of the philosophical question based on the conversation between meno and socrates 7. Suppose a force of 15 N is required to stretch and hold a spring 0.25 m from its equilibrium position. a. Assuming the spring obeys Hooks law, find the spring constant k. b. How much work is required to compress the spring 0.2 m from its equilibrium position Mikal is proving that AE CE . Triangles A B E and C D E are connected at point E. Angles A B E and E D C are right angles. The lengths of A C and D C are congruent.Which reason does the represent in Mikals proof?SASSSSASAAAS I WILL MARK BRAINLIEST PLEASE HELP !! Determine whether the two triangles are similar. 4= 3/7(3x+14)Solve for x in simplest form. A law used to finance public services, such as waste treatment plants, parks, and schools, in newly developed areas, can result in extra-high taxes, in addition to the normal property taxes, and MUST be made known to any buyer before a purchase takes place. This law is: 29Select the correct answer.How did Hitler use anti-Semitism to increase his power?O.A. He convinced Germans that Jews were responsible for the fallure of the National Socialist movement.OB. He fed German bellef that Jews were responsible for the loss of World War I, which caused the ills of the country.OC. He caused Germans' bellef that Jews were a threat to national security because they did not believe in domination of the world.OD. He gained the support of Germans with disabilities because of the Jews' bellef that people could only be strong if they were"pure."Reset Westfalia is a country with a conservative population that bases its culture around traditional values and long cultural traditions. This year Westfalia began to have an increase of electronic media, movies and literature from a neighboring country, Dogoni. Dogoni's culture is broadly different from Westfalia being more liberal and explicit in its media. Westfalia's parliament is now passing new protectionist policies to restrict or ban electronic media, movies and literature from Dogoni. What reason would most rationally support Westfalia's government actions what is your favorite scent at bath and body works? trying to rid my account of points. 1235 6 789Luis is researching how he can gain muscle. He hascome across the website www.protein4U.comPROTEIN 4 UProtein 4 U is a fast-acting and natural protein powderdesigned to help you gain muscle. It is pleasanttasting and dissolves easily into water or a favoritebeverage. Added to a weight-lifting or bodybuildingroutine, Protein 4 U will help you build muscle quicklyand safely. But don't listen to us-listen to what someof our customers are saying"Protein 4 U is the best product ever. It helped megain musclelMark this and return"The results were even faster than promised. In justtwo weeks, I started noticing a difference!"1 add it to my smoothie every day before my workout.I've tried protein shakes before, but this tastes and10Which statement is a correct evaluation of the source?O The source is valid because it contains informationthat comes directly from the company.O The source is invalid because it relies on personaltestimonials about the product.TIME REMAINING39:51O The source is valid because it cites the people whosupplied information for the article.O The source is invalid because it is unrelated to thetopic that Luis is researchingSave and ExitNextSubmit HUD received a complaint from Jameel, an Annapolis resident who claimed that his landlord evicted him when she found out he was gay. What will HUD likely do with this complaint Use the following set of data: 78, 78, 83, 85, 89, 91, 95, 98 what is the median for the set of data? (a) 85 (b) 89 (c) 87 (d) 98 please help!!!!!!!!!!!!!1 Your cousin asks you to help her prepare for her computer science exam next week. She gives you two clues about a type of relational database key: No column values may be NULL, and columns must be necessary for uniqueness. Which type of key is she referring to An astronaut is on the moon. He drops a hammer from a height of 3.2metres and it takes 2.0 seconds to reach the lunar landscape. What is the acceleration due to gravity of the moon? A vector of magnitube 5 units is inclined at an angle 37 to the x-axis and another vector of magnitube 10 units is inclined at 53 to the x-axis. What is the magnitude of the sumbof the vector components ( i ) along the x-axis ( ii ) along the y-axis? ( sin 37 = cos 53 = 0.6 / sin 53 = cos 37 = 0.8 ) The graph shows the heights, y (in centimeters), of a plant after a certain number of weeks, x. Donna drew the line of best fit on the graph.A graph titled Plant Height shows Number of Weeks on x axis and Height of Plant in cm on y axis. The scales on both x and y axes are shown from 0 to 5 at increments of 5. The graph shows dots at the ordered pairs 0, 1 and 0.5, 1.5 and 1, 2 and 1.5, 2.5 and 2, 2.8 and 2.5, 3 and 3, 3.4 and 3.5, 3.5 and 4, 4 and 4.5,4.5 and 5, 5. A straight line joins the ordered pairs 0, 1 and 5, 5What would most likely be the approximate height of the plant after 8 weeks? 11.0 centimeters 9.25 centimeters 8.8 centimeters 7.4 centimeters(Please show your work)Thank you! Mohandas gandhi used his philosophy of nonviolent noncooperation in an effort to:.